UTNianos

Versión completa: [Ejercicio 5] Duda problema de Limite de guia
Actualmente estas viendo una versión simplificada de nuestro contenido. Ver la versión completa con el formato correcto.
Hola gente como va?

Bueno practicando para el parcial, me surgio un problema con un ejercicio basico de limite que dice:

Sea \[f(x)=\left \{ 1\] si \[x\in \mathbb{Q}\]

\[f(x)=\left \{ -1\] si \[x\in \mathbb{I}\]

a) Demostrar utilizando la definicion de limite que: \[x\overset{lim}{\rightarrow}0f(x)\neq 1\]

b) Demostrar utilizando la definicion de limite que: \[x\overset{lim}{\rightarrow}0f(x)\neq -1\]

¿Existe \[l\in \mathbb{R}/x\overset{lim}{\rightarrow}0f(x)=l\]

Eso es todo. Saludos y muchas gracias ;D.
Creo que es así:

\[0 \; \epsilon \; \mathbb{Q}\]

\[\lim_{x \to 0} \frac{f(x)-f(0)}{x-0}=\lim_{x \to 0} \frac{f(x)-1}{x}=\frac{\to 0}{\to 0}\]

Aplico L'Hopital...

\[\lim_{x \to 0} \frac {f'(x)}{1}=f'(0)=0\]

No estoy muy seguro, fijate.

Saludos!
(10-08-2012 11:57)matyary escribió: [ -> ]Creo que es así:

\[0 \; \epsilon \; \mathbb{Q}\]

\[\lim_{x \to 0} \frac{f(x)-f(0)}{x-0}=\lim_{x \to 0} \frac{f(x)-1}{x}=\frac{\to 0}{\to 0}\]

Aplico L'Hopital...

\[\lim_{x \to 0} \frac {f'(x)}{1}=f'(0)=0\]

No estoy muy seguro, fijate.

Saludos!

Maty: Esa es la definicion de derivada basicamente jeje y no veo porque aplicarla en el ejercicio (mas cuando este esta dentro del capitulo de limites).

Gracias igualmente.
JAJAJAJAJA tenés razón... cualquier cosa escribí. Perdón.
(10-08-2012 13:04)matyary escribió: [ -> ]JAJAJAJAJA tenés razón... cualquier cosa escribí. Perdón.

No loco, ta todo bien jaja xD
¿Y cómo sería?

Racionales son todos los enteros que se pueden escribir de la forma \[\frac{p}{q}\] con \[q \neq 0\] e Irracionales los que no.
El \[0\] sería racional. Por ende el límite buscado tendría que valer \[1\]. Pero pide demostrar lo contrario. ¿Cómo? Lo estoy procesando...

Está difícil eh.
(10-08-2012 13:28)matyary escribió: [ -> ]¿Y cómo sería?

Racionales son todos los enteros que se pueden escribir de la forma \[\frac{p}{q}\] con \[q \neq 0\] e Irracionales los que no.
El \[0\] sería racional. Por ende el límite buscado tendría que valer \[1\]. Pero pide demostrar lo contrario. ¿Cómo? Lo estoy procesando...

Está difícil eh.

Verdad que si lo esta Confused. Mientras tanto voy a hacer otra pregunta, de otro ejercicio jeje.

Saludos y si pensas algo, avisame porfa =)
URLs de referencia